LSAT and Law School Admissions Forum

Get expert LSAT preparation and law school admissions advice from PowerScore Test Preparation.

 Administrator
PowerScore Staff
  • PowerScore Staff
  • Posts: 8917
  • Joined: Feb 02, 2011
|
#26986
Complete Question Explanation

Justify the Conclusion—SN. The correct answer choice is (D)

This conditional Justify question concludes that it is unlikely that real wages will increase significantly in the near future. This is based on the idea that for real wages to increase significantly productivity must increase notably, and currently this country’s businesses are investing very little in new technology (and this pattern is likely to continue). Clearly what must be connected here is the idea that if a country invests very little in new technology (or does not invest a lot in new technology) then productivity will not increase notably. This can be diagrammed as follows:
  • Invest heavily new tech ..... :arrow: ..... Productivity increase ..... :arrow: ..... Real wages increase significantly
(The bolded portion is the connection needed in the correct answer choice.)

Answer choice (A): The conclusion is about the near future, so the trends of the past are unrelated to the argument here.

Answer choice (B): You cannot know that a number of workers will acquire the skills to use whatever new technology may develop, so this answer choice cannot be thought to prove the conclusion.

Answer choice (C): Again, the conclusion is about the “near future” so this answer choice is not related to it (“sooner or later” is not the same as the “near future,” and if it were this answer choice would actually attack the conclusion).

Answer choice (D): This is the correct answer choice. If productivity increasing is necessary for wages to increase, and productivity requires substantial investment in new technology (which is not happening), then real wages cannot increase in the near future.

Answer choice (E): Even if the new technology that is being invested in is not yielding an increase in productivity, that is not enough to conclude that productivity in not increasing for other reasons. So even with this answer choice it is possible that the conclusion is incorrect (if productivity could still be increasing for other reasons, then real wages could still increase in the near future and the conclusion may not be true).
 Blueballoon5%
  • Posts: 156
  • Joined: Jul 13, 2015
|
#19311
I got the right answer for this through a process of elimination, but I am still uncertain about the answer explanation on the student online center.

1.) The student online center explains that, not investment in tech --> no productivity increase --> no wage increase. i don't understand how we can get this diagram. Based on the sufficient and necessary indicators in the stimulus, it appears as though the entire diagram should be reversed. Here is the diagram that I came up with:

Wage increase --> productivity increase
No wage increase --> no investment in tech

To connect these two diagrams, I used the contrapositive of the first. Then I got this long diagram: "No productivity increase --> no wage increase --> no investment in tech"

Could someone explain how this diagramming works in this stimulus because I think I'm missing something...

2.) In the student online center, the answer key explains (for answer choice D) that the productivity increase is in the necessary condition. But this is not the case. According to the answer key's diagram, the negation of productivity increase is the necessary condition.

Please help!
 Nikki Siclunov
PowerScore Staff
  • PowerScore Staff
  • Posts: 1362
  • Joined: Aug 02, 2011
|
#19332
Hi Blueballoon5%,

The first line of your diagram is correct, but the second one is not. Hence the problem you are having with this question.

When simplified, the stimulus has the following structure:
Premise 1: wage increase :arrow: productivity increase
Premise 2: NO tech investment

Conclusion: NO wage increase
Obviously, there is a gap in the logic here - the second premise came out of nowhere, and its relevance to the conclusion is missing. The question asks us to justify the conclusion, so we need to connect the dots. In particular, we need to establish that the lack of investment in new tech will somehow prevent real wages from going up. One way to do that would be to say that:

Justify Formula: NO tech investment :arrow: NO productivity increase

If true, then by the contrapositive of the first premise we can prove that wages will not increase:

NO tech investment :arrow: NO productivity increase :arrow: NO wage increase

This prephrase is consistent with answer choice (D).

Hope this helps!
 olafimihan.k
  • Posts: 25
  • Joined: Jul 04, 2017
|
#40033
Hello!

I picked answer choice E because when I did the negation technique, I came up with something like:

"The new technology in which businesses are currently investing IS contributing to an increase in technology."

Did I do this right, and if so wouldn't that then kill the argument?

Thank you
User avatar
 Dave Killoran
PowerScore Staff
  • PowerScore Staff
  • Posts: 5852
  • Joined: Mar 25, 2011
|
#40040
olafimihan.k wrote:Hello!

I picked answer choice E because when I did the negation technique, I came up with something like:

"The new technology in which businesses are currently investing IS contributing to an increase in technology."

Did I do this right, and if so wouldn't that then kill the argument?

Thank you

Hi O,

Thanks for the question! There's one big problem with using the Assumption Negation Technique here: it's for Assumption questions only, and this is a Justify question. So, the technique is inapplicable here (this is the case despite the fact that on occasion it will work on a Justify question; the problem is that it doesn't work all the time on Justify Qs).

That said, your negation of (E) was done correctly, so good job there! It still wouldn't kill the argument because the argument allows for degrees of increase and investment through the use of terms like "increases notably" and "investing very little." So, those statements allow for the new technology to contribute to an increase in productivity, and faced with the negation of (E), the author would reply, "Sure, but it's still too little—they need to invest more."

Please let me know if that helps. Thanks!
 olafimihan.k
  • Posts: 25
  • Joined: Jul 04, 2017
|
#40194
My mistake, thank you!
 lsatnoobie
  • Posts: 52
  • Joined: Sep 18, 2017
|
#42268
Hi Powerscore,

After diagramming the stimulus, I can logically arrive at answer choice D. However, I am still having a hard time ruling out answer choice E. One of your explanations state E is wrong because even if the new technology is not contributing to productivity, other things can be increasing the productivity.

Can’t this same argument be applied to D?

Through diagramming, is there a logical way I can conclude E is incorrect?

Thanks!
 Adam Tyson
PowerScore Staff
  • PowerScore Staff
  • Posts: 5153
  • Joined: Apr 14, 2011
|
#42303
Glad to help, lsatnoobie (a username that will soon be inaccurate, amiright?), but I will do so without diagramming answer E because it is not a conditional statement and therefore really doesn't lend itself to being diagrammed. That's part of the problem with it, actually - we need something to fill in the gap in a conditional chain, and only another conditional claim can do that! Since E is not conditional in itself, making the premises sufficient for the conclusion, it cannot be used to justify this conclusion.

Now, from a non-diagrammatic standpoint, let's take a look at a crucial issue in the stimulus - the gap. The author's premise is that we are not investing much in technology. His conclusion is that we will be unlikely to see substantial increases in productivity and thus no substantial increase in wages. The gap is between "not investing in tech" and "no significant gains in productivity", and what we need to close that gap and make the conclusion inescapable is that not investing is sufficient for not making gains, like so:

invest :arrow: gains

Does E do that? Well, it tells us that what little tech we are investing in isn't contributing to productivity, and that certainly strengthens the argument. But is that enough to prove that we won't get those gains? The conditional claim in D makes it a guarantee, because it makes a lack of investment sufficient for a lack of gains. In E, however, the investment isn't sufficient to prove anything. In other words, it doesn't make the lack of gains inescapable.

Here's an analogy for you: I am madly, deeply in love with Reese Witherspoon. She doesn't know it yet, but she is going to be my third wife. Maybe my fourth, we'll see what happens with Minnie Driver, but the thing is that she will not marry me unless I woo her first, and since I haven't brought her flowers and don't expect to any time soon (because her agent won't give me her address, no matter how many times I ask) she isn't likely to marry me in the near future.

There's a gap in my argument between "woo her" and "bring her flowers". How would I prove that she is not likely to marry me soon? Make flowers a necessary component of wooing her. I cannot woo her without flowers. If I woo her, I must bring her flowers. Flowers are required for wooing. That claim, if true, added to the original argument, would make the conclusion unavoidable. If flowers are required, and I have not and will not soon get flowers to her, then we shall not wed.

Would saying that my failed attempts so far to bring her flowers were insufficient for wooing her prove that she is unlikely to marry me soon? Or might she still marry me anyway, because I have wooed her in some other way? Perhaps the romantic gestures I have already made might be sufficient to win her heart and her hand? Maybe she prefers candy, or some ritual dance on her front lawn at midnight (I just need that address so I can try that one, darn it). She might see the light even absent the flowers, in which case my conclusion might prove false. It is not justified, which means it is not proven.

Wooing = Productivity Gains
Flowers - Investing in Technology
Marrying Reese = Wage Increase
Failed attempts at bringing flowers are insufficient to woo her = the tech we are investing in isn't increasing productivity

I hope my story helps shed the light not only on the problem with answer E, but on my love for Reese, and that someone reading this post will convey to her my feelings and help bring us together. Second only to my desire to help my students get higher LSAT scores, that is my highest wish. That, and that my current wife doesn't find out any of this.
 Blueballoon5%
  • Posts: 156
  • Joined: Jul 13, 2015
|
#57041
Nikki Siclunov wrote:Hi Blueballoon5%,

The first line of your diagram is correct, but the second one is not. Hence the problem you are having with this question.

When simplified, the stimulus has the following structure:
Premise 1: wage increase :arrow: productivity increase
Premise 2: NO tech investment

Conclusion: NO wage increase
Obviously, there is a gap in the logic here - the second premise came out of nowhere, and its relevance to the conclusion is missing. The question asks us to justify the conclusion, so we need to connect the dots. In particular, we need to establish that the lack of investment in new tech will somehow prevent real wages from going up. One way to do that would be to say that:

Justify Formula: NO tech investment :arrow: NO productivity increase

If true, then by the contrapositive of the first premise we can prove that wages will not increase:

NO tech investment :arrow: NO productivity increase :arrow: NO wage increase

This prephrase is consistent with answer choice (D).

Hope this helps!
Thank you! Going back to this question and reading your explanation, I understand now :)
User avatar
 ridolph.lauren
  • Posts: 16
  • Joined: Feb 20, 2022
|
#96691
As I am trying to review the correct answers I am confused as to why some Justify questions have "SN" next to them. This might be a dumb question or a concept I missed but what does "SN" stand for?

Thank you!

Get the most out of your LSAT Prep Plus subscription.

Analyze and track your performance with our Testing and Analytics Package.